LSAT and Law School Admissions Forum

Get expert LSAT preparation and law school admissions advice from PowerScore Test Preparation.

 jmramon
  • Posts: 47
  • Joined: Jul 21, 2017
|
#41841
So I realize I should be getting the first 10 correct and usually do, but this question tripped me up.

Is answer A not right because the stimulus is only referring to "this [particular] generalization" being regarded as scientific law despite the absence of verification and doesn't say that is the case with all scientific laws?

Why is D correct, though? How do we know that they tested the theory to the extent current science allows? The stimulus only says none of their tests falsified the theory and that they didn't test the theory under every feasible condition.

I'm concerned that this is supposed to be an easy question and I'm not seeing why D is right. B (opposite answer), C (stimulus doesn't claim this, it just the reverse and doesn't generalize from it), and E (not stated in stimulus) seem clearly wrong though. Thank you for the help ahead of time!!
 James Finch
PowerScore Staff
  • PowerScore Staff
  • Posts: 943
  • Joined: Sep 06, 2017
|
#41993
Hi JM,

Good job getting down to only a pair of Contenders and eliminating the rest as losers. Let's look at what the stimulus is arguing and then look at the difference between answer choices (A) and (D).

The argument the lecturer makes is that scientists cannot disprove a particular hypothesis under any available conditions, despite the acknowledgement that this hypothesis has not and currently cannot be tested under every possible condition. Despite this, the lecturer concludes that the hypothesis is correctly regarded as true (a scientific law). The question stem then asks us to justify the reasoning by adding a principle as a new premise.

Prephrasing the answer, we can see that there is a leap of logic between not being able to disprove something with the available methods and believing so strongly that that something is true as to enshrine it as a scientific law. So our principle has to involve anything that cannot be falsified/is confirmed with current methods being scientific law, or restated, if current methods cannot falsify a hypothesis, then that hypothesis should be regarded as scientific law.

Answer choices (A) and (D) both deal directly with this, but from opposite directions. (A) states that if it is a scientific law, it has not been falsified, making scientific law the sufficient and lack of falsification the necessary condition. This is a Mistaken Reversal of the reasoning we need to justify the stimulus's conclusion, and thus incorrect.

(D) uses confirmation (effectively lack of falsification) as the sufficient condition, and "considered a scientific law" as the necessary condition, fitting in exactly with our prephrase. One hint that this is the correct answer, and not (A), is the scope: this deals with "considered scientific law" which matches more exactly with "regarded as scientific law" that we have in the stimulus's conclusion.

Hope this clears things up!
 jmramon
  • Posts: 47
  • Joined: Jul 21, 2017
|
#42001
Thank you for the clarification, James. How embarrassing that I didn’t see this is a mistaken reversal; I should know better by now. I will do more conditional problem types so I don’t make this same error on test day! Thanks again!

Get the most out of your LSAT Prep Plus subscription.

Analyze and track your performance with our Testing and Analytics Package.